LSAT and Law School Admissions Forum

Get expert LSAT preparation and law school admissions advice from PowerScore Test Preparation.

User avatar
 Dave Killoran
PowerScore Staff
  • PowerScore Staff
  • Posts: 5853
  • Joined: Mar 25, 2011
|
#88046
Complete Question Explanation
(The complete setup for this game can be found here: lsat/viewtopic.php?f=165&t=7597)

The correct answer choice is (B).

The question stem specifies that Sukanya receives two messages from L. This means that either five or six messages must have been sent. The question stem also requests you to identify the greatest number of messages between L’s first message and L’s second message. Given that the idea is to maximize the number of messages between L’s two messages, the template containing six messages is more likely to yield the answer than the template containing five messages because there are more messages to work with.

Both Template 3A and 3B allow for a maximum of one message to be sent between L’s two messages:

G2-Q12-d1.png

Hence, the answer must be at least one, and answer choice (A) can be eliminated.

The templates containing five messages at best allow for no messages between L’s two messages:

G2-Q12-d2.png

Accordingly, the maximum is one, and answer choice (B) is correct.
You do not have the required permissions to view the files attached to this post.

Get the most out of your LSAT Prep Plus subscription.

Analyze and track your performance with our Testing and Analytics Package.